Author Topic: EM Drive Developments - related to space flight applications - Thread 2  (Read 3321417 times)

Offline TheTraveller

From the thermal and A/m plots from EW, most of the resonance is happening at the big end. I would not put the magnetron in that space because the input there will probably perturb the waves. Shawyer put the input near the small end. I would put it "at" the small end, depending on wave polarization. The walls should do most of the reflecting, not the small end.

Todd D.

Please take a gander at this Demo Drive by Shawyer: http://forum.nasaspaceflight.com/index.php?action=dlattach;topic=36313.0;attach=829850

It seems to have the feed near the big diameter end.

Good call doctor. I find the stub coupler/feed at the larger diameter of the cavity interesting...thinking this was  the end with the highest return loss/standing wave at 2.4 GHz...perhaps not.

The stepper motor appears to be adjusting the length of a tuning stub centered in the cavity on the narrow end, probably something like this: http://i.stack.imgur.com/Vfdxq.png Its and old tried and true methodology.

Regardless, this tuning stub is simply a matching element which can be fixed (non-adjustable) once a center frequency is set and a tuning stub length measurement can be made. Normally, this tuning stub is adjusted for best S parameter match/bandwidth: http://www.antenna-theory.com/definitions/sparameters.php

Thank you.  I agree with you   :)

If that stub extends inside the frustum, that would explain why he put the input at the big end. I still say, it should be at the small end to avoid perturbing the harmonics.

Todd

Somebody else pointed out that Shawyer's Demonstrator was feed via a waveguide instead of via coax.  Does that make any difference to your point that feeding should preferentially occur (if possible) at the small end to avoid perturbing harmonics? In other words do you think that feeding with a waveguide avoids perturbing harmonics and therefore if one feeds with a waveguide (fed upstream from a magnetron) you could just as well feed the waveguide at the big end ?
Shawyer feed it with a magnetron as that was what was available and what he used on the 1st test of principal EM Drive (attached) which was fed via a waveguide from the middle. The coax connector is for sense. You can see the waveguide behind the EmDrive.

I suspect due to the tuning latency issues, in the Flight Thruster he changed to a fixed cavity and using Rf from a narrow band variable frequency Rf generator that fed a TWTA and was quickly adjusted from the E field sensor installed in the big end of the cavity.
« Last Edit: 05/12/2015 09:57 pm by TheTraveller »
It Is Time For The EmDrive To Come Out Of The Shadows

Offline TheTraveller

There's still the issue of drifting off tune with temperature

... and acceleration. That is why I believe now that it should be pulsed, not steady state operation.

Todd
That also makes sense to me. 

Separately, but interestingly the Serrano Field Effect Boeing Darpa device tested by Dr. White displayed the highest thrust/InputPower of any device, yet it only showed very short time impulses (like Dirac Delta Functions) instead of steady state operation (although to me its principle of operation is very different from the EM Drive, Dr. White classified this device also as a Q-thuster).

This is the text for Boeing/DARPA in slide 40 of Dr. White's presentation (http://ntrs.nasa.gov/archive/nasa/casi.ntrs.nasa.gov/20140000851.pdf ):

<<SFE Test Article at JSC

In 2013, Boeing/DARPA sent Eagleworks Lab an SFE test article for testing and evaluation

Evaluation of the test article in and out of a Faraday Shield performed from Feb through June 2013.

• There is a consistent transient thrust at device turn-on and turn-off that is consistent with Qthruster physics
• The magnitude of the thrust scaled approximately with the cube of the input voltage (20-110uN).
• The magnitude of the thrust is dependent on the AC content of the turn-on and turn-off pulse
• Specific force of transient thrust was in the ~1- 20 N/kW range.

~20-110 uN Thrust Pulses
Specific Force ~1-20N/kW>>

NASA Eagleworks also provided this information in a 2013 Newsletter, which is available in the Internet from this link:  https://xa.yimg.com/kq/groups/86787010/513081407/name/Eagleworks+Newsletter+2013.pdf

that reads:

<<NASA/Boeing/SFE Campaign: Boeing/DARPA sent Eagleworks Lab an SFE test article for testing and
evaluation. The guest thruster was evaluated in numerous test configurations using varying degrees of
Faraday shielding and vacuum conditions. Observations show that there is a consistent transient thrust
at device turn-on and turn-off that is consistent with Q-thruster physics. The magnitude of the thrust
scaled approximately with the cube of the input voltage (20-110uN). The magnitude of the thrust is
dependent on the AC content of the turn-on and turn-off pulse. Thrust to power of transient thrust was
in the ~1-20 N/kW range


Yes, that's twenty Newtons per kiloWatt on the upper range
Have spoken with Hector Serrano. The SFE device EW tested produced torque pulses, not linear thrust. He is currently testing a linear version.
« Last Edit: 05/12/2015 10:01 pm by TheTraveller »
It Is Time For The EmDrive To Come Out Of The Shadows

Offline rfmwguy

  • EmDrive Builder (retired)
  • Senior Member
  • *****
  • Posts: 2205
  • Liked: 2713
  • Likes Given: 1134
good comments re tuning; fc (ctr freq) or mechanical (cavity). Thermal coeffecient of copper aside, another tuning requirement comes to mind...in a vacuum, the dielectric constant will change in the frustum, albeit small, air and vacuum differ http://hyperphysics.phy-astr.gsu.edu/hbase/tables/diel.html

I did not find thread discussion abt whether the frustum cavity was sealed during recent vac test...apologies if I overlooked one

Offline Jim Beichler

  • Member
  • Posts: 1
  • Ohio, USA
  • Liked: 0
  • Likes Given: 0
As far as I can tell, this propulsion system works by utilizing Dark Energy. That is according to my version of Einstein's unified theory which actually puts mach's Principle into a workable theoretical model. See the section on predictions in "The Einstein unified field theory completed: A direct challenge to the basic assumptions, theories and direction of modern and post-modern physics"
http://tinyurl.com/o2e62jb
« Last Edit: 05/13/2015 01:15 am by Chris Bergin »

Offline jmossman

  • Member
  • Posts: 73
  • San Jose, CA
  • Liked: 58
  • Likes Given: 180
...
I know infinite values are impossible. Likewise negative Dfs. That was not the point.

The point is the Df equation, applied to a variable frequency, shows there is an ideal frequency that will generate the best cavity Df. Driving the cavity with some chosen frequency, like 2.45GHZ will probably NOT make anything happen. Like trying to drive a tuned LC circuit with a frequency far away from it's resonate frequency. Waste of time. Likewise driving the cavity with the calculated best Df frequency will probably do the same thing. No thrust in the real world.

What the exercise shows is that the Rf generator driving the cavity should be operating 2x or 3x the best Df cavity frequency and that Rf frequency generating system must be able to vary the driving frequency so to search for the best frequency in the real world. The spreadsheet give me a starting place and an understand the cavity best Df frequency should be 1/2 or 1/3 the applied Rf frequency.

To me as an engineer starting a replication of the Flight Thruster and associated variable Rf generation system, it is very new and valuable information. This is all related to real world engineering (building actual hardware) to give the best chance of generating thrust.

To assisting theory development, well it may not be of much value.

As to why Shawyer is using a particular excitation frequency I suggest that you use your spreadsheet to check the above vs. the calculated natural frequencies and mode shapes here:

http://forum.nasaspaceflight.com/index.php?action=dlattach;topic=36313.0;attach=634741

(see in the slide the drawing insert detail containing the two dimensions (the diameter of the big base and the length of the truncated cone) given by Shawyer while the third dimension (the diameter of the small base) of the truncated cone is parametrized on the horizontal axis (ranging from a pointy cone to a cylinder) to ascertain its correct value)

Shawyer (to my knowledge) has never provided all three geometrical dimensions of the truncated cone.  Hence one of the dimensions (the diameter of the small base) has to be estimated (obtained from the inverse expression for the Design Factor -when Shawyer has provided the Design Factor, parametrized as per the attached file, or estimated from images, as done by @aero and others).  Since Shaywer has not provided all three dimensions, there is uncertainty as to what he actually did and why

@aero had correspondence with Shawyer asking for the dimension of the small diameter, to my recollection Shawyer cryptically answered "small base diameter chosen on the basis of the cut-off frequency" (hence still unknown how close to the cutoff wavelength was Shawyer's chosen small base diameter).

Be gentle as the following is highly speculative, but I’ve attempted to be “self-consistent” with several of the observed behaviors and characteristics of the Shawyer, Yang, and CW experiments.  In many ways the following is recirculating an idea already proposed within this forum, but putting a slightly different spin on it.

What if the standing wave within the frustum is necessary but not sufficient for thrust generation?

@TheTraveller’s recent observations about Shawyer’s design equation seem to be consistent with the resonant mode (i.e. standing wave) not being the primary source of thrust.  If the frustum is designed for more than just resonant mode characteristics, this could explain why Shawyer seems to be progressively increasing his frustum angle to much larger values than seem to be beneficial for a resonant cavity.  Perhaps the best empirical confirmation that resonance alone being insufficient for thrust involves a basic sanity check: 

Why has no one noticed superconducting resonant cavities floating or ripping themselves apart due to supposed EM drive thrust effects? 
Answer:  no EM drive thrust possible within a symmetrical resonant cavity


As postulated much earlier within this Forum's thread by others, the “standing wave necessary but not sufficient” conjecture would also help explain the disparity in thrust between the EW tests (a near pure standing wave at the cavity’s resonant frequency) versus the “dirty” magnetron sources in both Shawyer and Yang tests.

A follow-on to this line of reasoning would be to suspect that most (if not all) of EW’s vacuum-tested 40micronewton “thrust” is actually due to a combination of thermal effects altering center-of-mass and possible outgassing from the FR4 and/or dielectric insert.  (Although perhaps the non-resonant/off-frequency portions of the EW energy allows for marginal thrust, along with the possibility that the dielectric attenuation helps provide a gradient that somehow manifests as “thrust”)

Shawyer has described peculiar generator and motor modes for the EM drive.  While I still can’t wrap my head around the “motor” mode, the “generator” mode seems a bit less of a conceptual jump since we know the standing wave within the frustum is a source of stored energy.  If we start accelerating the frustum then there will be a red shift for the photons striking the end plate moving “away” from the rest frame;  this is analogous to a gravity induced red shift.  If the photons are close enough to the frustum cut-off frequency, the red shift will induce attenuation that is greater than the vanilla ohmic losses in a resonant cavity at rest.  How the attenuation results in a “thrust” is beyond me…  but the number of photons near the cut-off frequency will get larger based upon the cavity Q.  The larger the Q, the larger the number of photons that can be red-shifted into attenuation in response to acceleration (and presumably providing “thrust” in an attempt to counter said acceleration when in "generator" mode).  A similar thought experiment applies to the opposite end plate moving "towards" the rest frame, which should introduce a blue shift.  How does blue or red shifting correlate to "thrust"? Or is such an observation merely another Red Herring?  I have no idea...

I leave with this final thought:  what if the standing wave provided a “structure” within the cavity against which attenuated waves could interact?  I’ve attached a very crude Photoshopped image as an illustration….  I realize this conjecture should probably be best left to science fiction literature than this forum…. 

Perhaps the forum readership should just consider this post as another comedic break.   :P

Thanks,
James

Offline Razel.Korr

  • Member
  • Posts: 1
  • Primax -408.24 Epsilon
  • Liked: 0
  • Likes Given: 0
Obligatory Disclosure - I am not a trained professional in any way. I have been lurking this thread for the last couple months and I am entirely incapable of formulating my ideas mathematically.  However -

How can the principle of operation of the ring laser gyroscope be compared to the principle of operation of EM Drive thrust, escapes me...

This seems important. The Gyroscope uses lasers  sent in different directions and then their varying travel times due to rotation to create the interference. The EMD [supposedly?] uses the difference between speeds at two ends of a wave to generate an effect. Both items [EMD and LRG] use propagating waves being affected by differences in their velocities to create their results. Would that I could dig through the equations for a jumping off point.

Offline deltaMass

  • Full Member
  • ****
  • Posts: 955
  • A Brit in California
  • Liked: 671
  • Likes Given: 275
Serrano's 0.02 N/W means breakeven speed is only 100 m/s = 2/k

Offline TheTraveller

Serrano's 0.02 N/W means breakeven speed is only 100 m/s = 2/k
According to Serrano, the device EW tested only produces rotary torque forces, not linear force. So could spin a spacecraft around it's CG but can't alter its velocity.
It Is Time For The EmDrive To Come Out Of The Shadows

Offline WarpTech

  • Full Member
  • ****
  • Posts: 1407
  • Do it!
  • Statesville, NC
  • Liked: 1453
  • Likes Given: 1925
There's still the issue of drifting off tune with temperature

... and acceleration. That is why I believe now that it should be pulsed, not steady state operation.

Todd
That also makes sense to me. 

Separately, but interestingly the Serrano field effect device tested by Dr. White displayed the highest thrust/InputPower of any device, yet it only showed very short time impulses (like Dirac Delta Functions) instead of steady state operation (although to me its principle of operation is very different from the EM Drive, Dr. White classified this device also as a Q-thuster).
Forgive me for interrupting, but with pulsing, do you mean Pulse Width Modulation or PWM?

Yes. Ramp it up to peak thrust and let it exponentially decay back to zero... and Repeat. Since it can't spend more than the input power supplies and remain in steady state operation, and we already know that the input power will only provide uN of thrust. Higher thrust cannot be sustained at steady state operation. Higher thrust levels can only be momentary, and will quickly decay to nothing when output exceeds input.

Todd

Offline WarpTech

  • Full Member
  • ****
  • Posts: 1407
  • Do it!
  • Statesville, NC
  • Liked: 1453
  • Likes Given: 1925
From the thermal and A/m plots from EW, most of the resonance is happening at the big end. I would not put the magnetron in that space because the input there will probably perturb the waves. Shawyer put the input near the small end. I would put it "at" the small end, depending on wave polarization. The walls should do most of the reflecting, not the small end.

Todd D.

Please take a gander at this Demo Drive by Shawyer: http://forum.nasaspaceflight.com/index.php?action=dlattach;topic=36313.0;attach=829850

It seems to have the feed near the big diameter end.

Good call doctor. I find the stub coupler/feed at the larger diameter of the cavity interesting...thinking this was  the end with the highest return loss/standing wave at 2.4 GHz...perhaps not.

The stepper motor appears to be adjusting the length of a tuning stub centered in the cavity on the narrow end, probably something like this: http://i.stack.imgur.com/Vfdxq.png Its and old tried and true methodology.

Regardless, this tuning stub is simply a matching element which can be fixed (non-adjustable) once a center frequency is set and a tuning stub length measurement can be made. Normally, this tuning stub is adjusted for best S parameter match/bandwidth: http://www.antenna-theory.com/definitions/sparameters.php

Thank you.  I agree with you   :)

If that stub extends inside the frustum, that would explain why he put the input at the big end. I still say, it should be at the small end to avoid perturbing the harmonics.

Todd

Somebody else pointed out that Shawyer's Demonstrator was feed via a waveguide instead of via coax.  Does that make any difference to your point that feeding should preferentially occur (if possible) at the small end to avoid perturbing harmonics? In other words do you think that feeding with a waveguide avoids perturbing harmonics and therefore if one feeds with a waveguide (fed upstream from a magnetron) you could just as well feed the waveguide at the big end ?

If the "Todd Conjecture" is correct, then we want the input momentum facing toward the large end. If the small end were made the same size as the waveguide feeder, reflected waves should not be able to travel back up the magnetron. All the reflections should be off the large end and the walls, and there is nothing to perturb the resonance. Injecting in the side or big end is likely to disrupt the resonance that we want to amplify.

The only difference between a waveguide feeder and a coax from what I can see, is the bandwidth of the signal. A magnetron has more energy over a wider bandwidth.

Todd

Offline PaulF

  • Member
  • Posts: 52
  • Netherlands
  • Liked: 19
  • Likes Given: 9
There's still the issue of drifting off tune with temperature

... and acceleration. That is why I believe now that it should be pulsed, not steady state operation.

Todd
That also makes sense to me. 

Separately, but interestingly the Serrano field effect device tested by Dr. White displayed the highest thrust/InputPower of any device, yet it only showed very short time impulses (like Dirac Delta Functions) instead of steady state operation (although to me its principle of operation is very different from the EM Drive, Dr. White classified this device also as a Q-thuster).
Forgive me for interrupting, but with pulsing, do you mean Pulse Width Modulation or PWM?

Yes. Ramp it up to peak thrust and let it exponentially decay back to zero... and Repeat. Since it can't spend more than the input power supplies and remain in steady state operation, and we already know that the input power will only provide uN of thrust. Higher thrust cannot be sustained at steady state operation. Higher thrust levels can only be momentary, and will quickly decay to nothing when output exceeds input.

Todd
Funny. I suggested PWM like 20-25 pages ago...  ::)

But to stay on topic, if PWM is used with microwaves, to my knowledge the wavelength of the microwaves would be altered by the short pulses, I forget the name of the effect, but it averages out to the wavelength with the energy of the full cycle (duty+null effort) (or so I presume). Is this a problem or a boon? This is where my knowledge reaches it's limits. I am presuming using low frequency duty cycles would negate the effect.
« Last Edit: 05/12/2015 11:54 pm by PaulF »

Offline frobnicat

  • Full Member
  • ****
  • Posts: 518
  • Liked: 500
  • Likes Given: 151
...
The larger the Q, the larger the number of photons that can be red-shifted into attenuation in response to acceleration (and presumably providing “thrust” in an attempt to counter said acceleration when in "generator" mode).  A similar thought experiment applies to the opposite end plate moving "towards" the rest frame, which should introduce a blue shift.  How does blue or red shifting correlate to "thrust"? Or is such an observation merely another Red Herring?  I have no idea...
...

About that there is a simple conventional idea : the "thrust" yielded by blue or red shifting of contained photons due to acceleration of container (relative to some inertial frame) is always opposite to acceleration, doesn't depend on the shape of the container, is proportional to mass equivalent E/c˛ of contained energy E (that is indeed proportional to Q, for a given power input), and is given by  Fphotons/container=-(E/c˛)*a (vectors in bold).

Conventional justification : to accelerate the container of mass mc implies also accelerating the mass equivalent of its content (photons) mp=E/c˛, one must provide an exterior force on container Fext/container=(mc+mp)*a. Insisting that what is to be accelerated is only the mass of container makes an artificial pseudo-force term to appear on the left :
Fext/container+Fphotons/container=mc*a with Fphotons/container=-mp*a

Hence it appears that this "thrust" is not a thrust as it will always subtract from an exterior force that tries to make the container accelerate. Actually this is just the opposing "inertial force" of the mass equivalent of energy content. Put more simply, and leaving things at their place in the dynamic equation, it's just that contained photons (fields, whatever) add mass to the system (container+contained).

My rough and clumsy estimates with a cavity of Q=10000 and input power of 100W was giving energy content of 10-3 J. Later (probably better) calculation by deltaMass would give more on the order of 10-4 J (for same Q=10000 @ 100W around 2GHz). Anyway, will take 10-3 J as an upper bound of contained EM energy at a given time within frustum tested at Eagleworks. This is equivalent to a mass on the order of 10-20 kg. Before it played a role at the µN level, opposite from thrust moreover, would mean accelerations (of the container) on the order of 1013 gees !

@WarpTech or other proponents of "dependence of the effect on acceleration", please run the numbers. What would be the acceleration of frustum needed to really drift out of bandwidth for Q around 10000 or otherwise reach a magnitude significant to behaviour of waves inside ?

Offline Corlock Striker

  • Member
  • Posts: 13
  • New York, NY
  • Liked: 8
  • Likes Given: 1
Why has no one noticed superconducting resonant cavities floating or ripping themselves apart due to supposed EM drive thrust effects? 
Answer:  no EM drive thrust possible within a symmetrical resonant cavity


I've been mostly lurking through the thread for the past few days, and I can't claim to be any sort of an expert on any of this.  I can only just barely follow what you all are discussing.  However, there's been a lot of talk about the potential for thermal artifacts to be causing the thrust, and jmossman made mention of super conductors in the post I quoted above.  I came across this article earlier today that I thought might be of interest to you all, dealing with a team from the Max Planck Institute creating a super conductor at room temperature for a fraction of a nanosecond in a ceramic material that had a small amount of copper in it.  Perhaps somehow the EM drive is achieving a similar effect?

http://www.sciencealert.com/physicists-achieve-superconductivity-at-room-temperature?utm_source=Article&utm_medium=Website&utm_campaign=InArticleReadMore

I just thought that might be something that would be useful to all of you.  Good luck in cracking this.

Offline rrb6699

  • Member
  • Posts: 5
  • Liked: 0
  • Likes Given: 0
is there any way to recapture the "thrust" out of
 the main part of the stream and recycle it (so to say) so it could be reflected back into the chamber so it can be used more efficiently?

rr

Offline Rodal

  • Senior Member
  • *****
  • Posts: 5911
  • USA
  • Liked: 6124
  • Likes Given: 5564
@ RODAL

Just got a minute but from your p expression;

If L1/c1 = L2/c2

del f = (1/2*f)*((c1*c2)/(L1*L2))*b^2*((1/dD1^2)-(1/dD2^2))

might be a solution ??

Got to check the thinking later.

Night !

I find your previous expression

del f = ( f/(2*c^2)) * (c1^2-c2^2)

more physically appealing, since it goes to zero for equal dielectric constants, regardless or their dielectric length,

while on the other hand

del f = (1/2*f)*((c1*c2)/(L1*L2))*b^2*((1/dD1^2)-(1/dD2^2))

goes to zero for equal dielectric lengths, regardless of their dielectric constants.

The previous expression is only valid approximation for a "uniformly varying dielectric".  There is no L1 and L2 in that case.

What do you think might maximize the second expression ? (valid only for L1/c1 = L2/c2 )

I was discussing this last night and we made some interesting observations. In a variable dielectric, like in the frustum, when the waves are accelerating to a higher group velocity, they are losing momentum. This momentum is lost to the material "in the direction of the wave". It is similar to frame dragging. The wave is losing energy trying to drag the waveguide or the dielectric with it.

After the wave is reflected, it again tries to drag the dielectric or frustum with it, and this time it meets more resistance. It becomes an evanescent wave and decays faster.

I do not believe a small end cap is needed and the frustum should taper all the way down to the wave guide feeding it. The reflected waves cannot reach the small plate. That's what the thermal images show as well. Most of the energy I think should be trapped at the big end.

Todd D.

Looking for a mechanical analogy :
Let's play with a bended pipe and a ball rolling in it. The pipe can constrain the ball to various path, it can rise or fall, at various steepness. Height of the pipe at a given location defines gravitational potential energy of the ball there. The ball is launched with a given velocity, and then turns around the pipe if it is a closed circuit, or goes back and forth if the two ends of the pipe are high enough, should make no difference.

Assuming no friction, the ball goes-on forever. When rising the ball loses kinetic energy, slows, and imparts momentum to the pipe. When on the return path (different part of pipe if circuit path or same part of pipe if going back and forth), the same delta height will make ball regain same kinetic energy as lost when rising, accelerate, and imparts momentum again. When taking curves, ball also imparts momentum on pipe. Integrating all those momentum exchanges on a cycle yields 0 net momentum. Not depending on path details.

Assuming a closed circuit path and friction (dry, viscous, magnetic... whatever dissipative interaction), including parts with low friction (forth) and parts with high friction (back) and arbitrary height profile (potential well whatever). After a number of cycles the ball will come to rest. Integrating all the momentum exchanges of ball on pipe (changes of height, curves, friction) will yield a total momentum equal to the initial momentum of the ball when launched. Not depending on path details and what parts are more or less dissipative.

I know a photon is not a ball but my question is, in "Newtonian layman's terms" how does the line of thinking you are developing making that analogy not valid, i.e. imply apparent deviation from conservation of momentum ?

The ball (photon) doesn't fall back down the well. There is nothing to give it back enough energy to do so. It dissipates in multiple reflections between the walls and the big end. They are not getting more out than they put in, so it does not violate conservation of energy. They are simply getting more NET momentum on one direction than in the other direction because there is more dissipation and attenuation in one direction than there is in the other. Dissipative systems are typically "not" conservative, loses prevent a true equal measure from occuring in both directions.

Todd D.

Come to think of it, it is not particularly surprising that a gradient is more easily interpreted in the comoving frame than in covariant form: dissipative phenomena are by nature alien to covariance.

They are associated with the production of entropy, they have a thermodynamical arrow of time.

It has taken some time for me to understand why Notsosureofit was using this approach but I'm slowly getting there   ;)

« Last Edit: 05/13/2015 01:10 am by Rodal »

Offline WarpTech

  • Full Member
  • ****
  • Posts: 1407
  • Do it!
  • Statesville, NC
  • Liked: 1453
  • Likes Given: 1925
...

@WarpTech or other proponents of "dependence of the effect on acceleration", please run the numbers. What would be the acceleration of frustum needed to really drift out of bandwidth for Q around 10000 or otherwise reach a magnitude significant to behaviour of waves inside ?

I read it in Shawyer's recent paper, he simulated it on a computer. I don't have that type of software. I've been wishing I did for decades. Since the system moves by attenuation and dissipation, the photons lose momentum and are red-shifted, while the frustum either gains momentum or gains heat from them.

You're correct, that the momentum it gains from the photons is in the direction of the photons, but results from the difference in the attenuation in each direction. The photons lose more momentum moving inward than moving outward, because they become evanescent waves. They do not increase their energy, except what they can take away from the frustum.

So I see it like ringing a bell. It is the exponential decay from a higher energy state that is giving the thrust. Attempting to make the Q very high to sustain resonance requires reducing the losses, but it is the losses that give it thrust. So... Shawyer increases the angle to make it more like a pill box. Anything over pi/6 is very close to a pill box. Then it should have a higher Q, but it should also have less efficient use of it.

If a photon rocket is: F/P = 1/c
and the Frustum is: F/P ~ Q/c x pulse width
Design efficiency should then target: (F*c)/(P*Q) = 1 but in practice < 1

This would imply maximizing thrust with a lower value of Q, i.e., we do not want to maximize Q, we want to maximize asymmetry in the attenuation, which is what I'm working on at the moment.

Best Regards,
Todd

 

Offline Notsosureofit

  • Full Member
  • ****
  • Posts: 691
  • Liked: 747
  • Likes Given: 1729
@ RODAL

Just got a minute but from your p expression;

If L1/c1 = L2/c2

del f = (1/2*f)*((c1*c2)/(L1*L2))*b^2*((1/dD1^2)-(1/dD2^2))

might be a solution ??

Got to check the thinking later.

Night !

I find your previous expression

del f = ( f/(2*c^2)) * (c1^2-c2^2)

more physically appealing, since it goes to zero for equal dielectric constants, regardless or their dielectric length,

while on the other hand

del f = (1/2*f)*((c1*c2)/(L1*L2))*b^2*((1/dD1^2)-(1/dD2^2))

goes to zero for equal dielectric lengths, regardless of their dielectric constants.

The previous expression is only valid approximation for a "uniformly varying dielectric".  There is no L1 and L2 in that case.

What do you think might maximize the second expression ? (valid only for L1/c1 = L2/c2 )

I was discussing this last night and we made some interesting observations. In a variable dielectric, like in the frustum, when the waves are accelerating to a higher group velocity, they are losing momentum. This momentum is lost to the material "in the direction of the wave". It is similar to frame dragging. The wave is losing energy trying to drag the waveguide or the dielectric with it.

After the wave is reflected, it again tries to drag the dielectric or frustum with it, and this time it meets more resistance. It becomes an evanescent wave and decays faster.

I do not believe a small end cap is needed and the frustum should taper all the way down to the wave guide feeding it. The reflected waves cannot reach the small plate. That's what the thermal images show as well. Most of the energy I think should be trapped at the big end.

Todd D.

Looking for a mechanical analogy :
Let's play with a bended pipe and a ball rolling in it. The pipe can constrain the ball to various path, it can rise or fall, at various steepness. Height of the pipe at a given location defines gravitational potential energy of the ball there. The ball is launched with a given velocity, and then turns around the pipe if it is a closed circuit, or goes back and forth if the two ends of the pipe are high enough, should make no difference.

Assuming no friction, the ball goes-on forever. When rising the ball loses kinetic energy, slows, and imparts momentum to the pipe. When on the return path (different part of pipe if circuit path or same part of pipe if going back and forth), the same delta height will make ball regain same kinetic energy as lost when rising, accelerate, and imparts momentum again. When taking curves, ball also imparts momentum on pipe. Integrating all those momentum exchanges on a cycle yields 0 net momentum. Not depending on path details.

Assuming a closed circuit path and friction (dry, viscous, magnetic... whatever dissipative interaction), including parts with low friction (forth) and parts with high friction (back) and arbitrary height profile (potential well whatever). After a number of cycles the ball will come to rest. Integrating all the momentum exchanges of ball on pipe (changes of height, curves, friction) will yield a total momentum equal to the initial momentum of the ball when launched. Not depending on path details and what parts are more or less dissipative.

I know a photon is not a ball but my question is, in "Newtonian layman's terms" how does the line of thinking you are developing making that analogy not valid, i.e. imply apparent deviation from conservation of momentum ?

The ball (photon) doesn't fall back down the well. There is nothing to give it back enough energy to do so. It dissipates in multiple reflections between the walls and the big end. They are not getting more out than they put in, so it does not violate conservation of energy. They are simply getting more NET momentum on one direction than in the other direction because there is more dissipation and attenuation in one direction than there is in the other. Dissipative systems are typically "not" conservative, loses prevent a true equal measure from occuring in both directions.

Todd D.

Come to think of it, it is not particularly surprising that a gradient is more easily interpreted in the comoving frame than in covariant form: dissipative phenomena are by nature alien to covariance.

They are associated with the production of entropy, they have a thermodynamical arrow of time.

That does suggest that one might be able to calculate the change in entropy between the "strained" photon distribution in the rest frame vs a "balanced" distribution in the accelerated frame.  A change in entropy can be related to a force (waves at seashore, shock fronts, etc)

I can vaguely remember that there were some theorems about that developed in studying the magnetosphere in the 70's.  I believe it was J. McGuire (or MacQuire? ) who worked w/ Van-Alan and Caravillano on that stuff ???

These are the right guys..a place to start:
Carovillano, R.L., and J.J. McGuire, Magnetic energy relationships
in the magnetosphere, in Physics of the Magnetosphere, edited
by R.L. Carovillano, J.F. McClay, and H.R. Radoski, pp. 290-300,
D. Reidel, Norwell, Mass., 1968

This the sort of thing:
http://qap2.onlinelibrary.wiley.com/doi/10.1029/RG011i002p00289/pdf

but, it doesn't seem to be immediately useful....seems there might be more about entropy in and about the theory of those shock fronts.... ?

Maybe Eq. 62 generalizing A1 and following ?  (62 looks worth remembering for future ref.)

Well, it was worth a shot.  The entropy calculation looks worth doing.  A difference would indicate an  irreversible process connected to the development of a force due to a photon distribution change of state.
« Last Edit: 05/13/2015 02:26 am by Notsosureofit »

Offline rfmwguy

  • EmDrive Builder (retired)
  • Senior Member
  • *****
  • Posts: 2205
  • Liked: 2713
  • Likes Given: 1134
...

@WarpTech or other proponents of "dependence of the effect on acceleration", please run the numbers. What would be the acceleration of frustum needed to really drift out of bandwidth for Q around 10000 or otherwise reach a magnitude significant to behaviour of waves inside ?

I read it in Shawyer's recent paper, he simulated it on a computer. I don't have that type of software. I've been wishing I did for decades. Since the system moves by attenuation and dissipation, the photons lose momentum and are red-shifted, while the frustum either gains momentum or gains heat from them.

You're correct, that the momentum it gains from the photons is in the direction of the photons, but results from the difference in the attenuation in each direction. The photons lose more momentum moving inward than moving outward, because they become evanescent waves. They do not increase their energy, except what they can take away from the frustum.

So I see it like ringing a bell. It is the exponential decay from a higher energy state that is giving the thrust. Attempting to make the Q very high to sustain resonance requires reducing the losses, but it is the losses that give it thrust. So... Shawyer increases the angle to make it more like a pill box. Anything over pi/6 is very close to a pill box. Then it should have a higher Q, but it should also have less efficient use of it.

If a photon rocket is: F/P = 1/c
and the Frustum is: F/P ~ Q/c x pulse width
Design efficiency should then target: (F*c)/(P*Q) = 1 but in practice < 1

This would imply maximizing thrust with a lower value of Q, i.e., we do not want to maximize Q, we want to maximize asymmetry in the attenuation, which is what I'm working on at the moment.

Best Regards,
Todd

Barely able to keep pace Todd...its a good thing. maximizing asymmetry in attenuation different from absorption like the stuff I used to work with?

http://www.westernrubber.com/products/himag-microwave-absorbers/himag-cavity-resonance-absorbers/

Offline SeeShells

  • Senior Member
  • *****
  • Posts: 2442
  • Every action there's a reaction we try to grasp.
  • United States
  • Liked: 3186
  • Likes Given: 2708
...

Best Regards,
Todd

Barely able to keep pace Todd...its a good thing. maximizing asymmetry in attenuation different from absorption like the stuff I used to work with?

http://www.westernrubber.com/products/himag-microwave-absorbers/himag-cavity-resonance-absorbers/

It is getting pretty hot and heavy in here and I'm not sure I am keeping up either. Love it though.
I've been mulling around the ideas of harmonics and wondered if anyone has considered injecting 2 RF sources into the cavity,
One set and the other variable in frequency? I've been slowly working my way through this but like I said it's been slow. I welcome and inputs and thoughts.

Offline rfmwguy

  • EmDrive Builder (retired)
  • Senior Member
  • *****
  • Posts: 2205
  • Liked: 2713
  • Likes Given: 1134
...

Best Regards,
Todd

Barely able to keep pace Todd...its a good thing. maximizing asymmetry in attenuation different from absorption like the stuff I used to work with?

http://www.westernrubber.com/products/himag-microwave-absorbers/himag-cavity-resonance-absorbers/

It is getting pretty hot and heavy in here and I'm not sure I am keeping up either. Love it though.
I've been mulling around the ideas of harmonics and wondered if anyone has considered injecting 2 RF sources into the cavity,
One set and the other variable in frequency? I've been slowly working my way through this but like I said it's been slow. I welcome and inputs and thoughts.

From what I gather a magnetron source is full of harmonics and subharmonics...one called it "dirty" which is a good visual... spectrum-wise.

Tags:
 

Advertisement NovaTech
Advertisement Northrop Grumman
Advertisement
Advertisement Margaritaville Beach Resort South Padre Island
Advertisement Brady Kenniston
Advertisement NextSpaceflight
Advertisement Nathan Barker Photography
1